RECIPROCATING ENGINE
OPERATION, MAINTENANCE,
INSPECTION, AND OVERHAUL
     This chapter includes the basic practices and tools used to inspect, repair, overhaul, and service recipro-
     cating engines. Also included is information on powerplant troubleshooting, removal, and installation, as
     well as maintenance practices used to inspect and service electrical and mechanical engine instruments.
2-1.            A01                                          2-1. Answer C. JSPT 2-13 (AC 65-9A)
A condition that can occur in radial engines but is          Before starting a radial engine that has been shut-down
unlikely to occur in horizontally opposed engines is         for more than 30 minutes, the propeller should be pulled
                                                             through by hand in the direction of normal rotation to
A alve overlap.                                              detect hydraulic lock. Hydraulic lock is more likely to
B zero valve clearance.                                      occur in radial engines because the inverted lower cylinders
C hydraulic lock.                                            may allow oil to collect in the combustion chambers.
                                                             Answer A is incorrect because valve overlap is a condition
                                                             where the intake and exhaust valves are open at the same
                                                             time to increase the volumetric efficiency of a cylinder.
                                                             Valve overlap is used extensively in all
                                                             reciprocating-engines. Answer B is incorrect because zero
                                                             valve clearance is commonly found in engines that use
                                                             hydraulic valve lifters which are commonly found in radial
                                                             and horizontally opposed engines.
2-2.             A02                                         2-2. Answer C. JSPT 2-14 (AC 65-12A)
If the intake valve is opened too early in the cycle of      The distance an intake valve may be opened before
operation of a four-stroke cycle engine, it may result in    TDC is limited by several factors. For example, if the
                                                             intake valve opens too early hot gases remaining in the
A    improper scavenging of exhaust gases.                   cylinder may flash back into the intake pipe and induc-
B    engine kickback.                                        tion system causing a backfire. Answer (A) is incorrect
C    backfiring into the induction system.                   because exhaust gas scavenging is accomplished
                                                             through the exhaust valve. Answer (B) is incorrect
                                                             because engine kickback is a function of faulty ignition
                                                             timing rather than valve timing.
2-3.            A02                                          2-3. Answer B. JSPT 2-54 (AC 65-12A)
On which part of the cylinder walls of a normally oper-      At the top of a stroke, a piston is subjected to extreme
ating engine will the greatest amount of wear occur?         heat, pressure, and a more erosive environment than at
                                                             the bottom of a stroke. These factors tend to cause
A    Near the center of the cylinder where piston            greater piston movement at the top of a cylinder.
     velocity is greatest.                                   Therefore, cylinder walls tend to wear more at the top
B    Near the top of the cylinder.                           than at the bottom (answer B).
C    Wear is normally evenly distributed.
2-2                                                                                                    Turbine Engines
2-4.            A02                                             2-4. Answer B. JSPT 2-55 (AC 65-12A)
During overhaul, reciprocating engine exhaust valves            Intake and exhaust valves can be checked for stretch
are checked for stretch                                         by one of two methods. One involves checking the
                                                                diameter of the valve stem near the neck of the valve
A with a suitable inside spring caliper.                        with a micrometer. If the diameter is smaller than nor-
B with a contour or radius gauge.                               mal, the valve has been stretched. The second method
C by placing the valve on a surface plate and mea-              involves checking the valve with a radius or contour
suring its length with a vernier height gauge.                  gauge. The contour gauge is designed to fit along the
                                                                underside of the valve head. If the contour of the gauge
                                                                and that of the valve do not match, it indicates that the
                                                                valve has been stretched. Answer (A) is wrong because
                                                                an inside spring caliper is used to measure inside
                                                                dimensions and answer (C) is wrong because a vernier
                                                                height gauge is not meant to be used for measuring
                                                                valve clearance.
2-5.             A02                                            2-5. Answer B. JSPT 2-41 (AC 65-12A)
During overhaul, the disassembled parts of an engine            Water-mixed degreasing compounds usually contain
are usually degreased with some form of mineral spirits         some form of alkali. If allowed to remain in the pores of
solvent rather than water-mixed degreasers primarily            metal engine parts when the engine is returned to ser-
because                                                         vice, the alkali will contaminate the oil and cause oil
                                                                foaming. Answer (A) is incorrect because some
A     solvent degreasers are much more effective.               water-mixed degreasers are just as effective as solvent
B     water-mixed degreaser residues may cause engine           degreasers. Answer (C) is wrong because, although
      oil contamination in the overhauled engine.               water-mixed degreasers can cause corrosion in alu-
C     water-mixed degreasers cause corrosion.                   minum and magnesium parts, it is not an absolute.
2-6.              A02                                           2-6. Answer C. JSPT 2-63 (AC 65-12A)
(1) Cast iron piston rings may be used in chrome-               Both statements (1) and (2) are true. As a general rule,
     plated cylinders.                                          chrome rings should never be used in a chrome cylin-
(2) Chrome-plated rings may be used in plain steel              der. However, it is extremely important that only
     cylinders.                                                 approved piston ring and cylinder combinations be
                                                                used. If approved combinations are not used, exces-
Regarding the above statements,                                 sive cylinder and/or piston ring wear could result.
A nly No. 1 is true.,
B neither No. 1 nor No.2 is true.
C both No. 1 and No.2 are true.
2-7.              A02                                           2-7. Answer A. JSPT 2-41 (AC 65-12A)
When cleaning aluminum and magnesium engine parts,              Water-mixed degreasing compounds usually contain
it is inadvisable to soak them in solutions containing          alkali or soap which, if allowed to remain in the pores of
soap because                                                    the metal, will react with hot oil and cause foaming.
                                                                Answer (B) is incorrect because soap does not chemi-
A     some of the soap will become impregnated in the           cally alter aluminum and magnesium. Answer (C) is
       surface of the material and subsequently cause           wrong because soap solutions do not cause dissimilar
       engine oil contamination and foaming.                    metal electrolytic action which can destroy engine parts
B     the soap can chemically alter the metals causing          in a matter of minutes. Water-mixed degreasing
       them to become more susceptible to corrosion.            solutions containing soap are potentially corrosive to
C     the parts can be destroyed by dissimilar metal            aluminum and magnesium engine parts only if the
       electrolytic action if they are placed together in the   parts are not rinsed thoroughly after cleaning.
       solution for more than a few minutes.
Turbine Engines                                                                                                 2-3
2-8.            A03                                         2-8. Answer B. JSPT 2-16 (AC 65-12A)
What is the purpose of a power check on a reciprocat-       The basic principle behind doing a power check is to
ing engine?                                                 measure the performance of an engine against an
                                                            established standard and determine if the engine is
A    To check magneto drop.                                 performing satisfactorily. Answer (A) is incorrect
B    To determine satisfactory performance.                 because a magneto check is used to check for the
C    To determine if the fuel/air mixture is adequate.      appropriate magneto drop and answer (C) is wrong
                                                            because the fuel/air mixture is checked during the
                                                            cruise mixture check.
2-9.            A03                                         2-9. Answer A. JSPT 2-22 (AC 65-12A)
Which of the following will be caused by excessive          When valve clearance is excessive, the valves will not
valve clearance of a cylinder on a reciprocating aircraft   open as wide or remain open as long as they should.
engine?                                                     Therefore, both the intake and exhaust valves will open
                                                            late and close early resulting in a reduced valve overlap
A    Reduced valve overlap period.                          period. Answer (B) is incorrect because the valves will
B    Intake and exhaust valves will open early and          open late and close early, not open early and close late.
      close late.                                           Furthermore, excessive valve clearances decrease
C    A power increase by shortening the exhaust             engine power output due to the reduced volumetric effi-
      event.                                                ciency caused by the reduced overlap period.
2-10.           A03                                         2-10. Answer A. JSPT 2-33 (AC 65-12A)
Which of the following would indicate a general             When performing a power check on a reciprocating
weak-engine condition when operated with a                  engine, it should be noted that with a constant air den-
fixed-pitch propeller or test club?                         sity, a given propeller and blade angle will always turn
                                                            at the same rpm for a given horsepower. Therefore, if
A    Lower than normal static RPM, full throttle oper-      an engine is producing a lower than normal rpm at a full
     ation.                                                 throttle setting, the engine may be weak. Answers (B)
B    Manifold pressure lower at idle RPM than at static     and (C) are incorrect because a low manifold pressure
     RPM.                                                   is often the result of early ignition timing and not
C    Lower than normal manifold pressure for any            necessarily an indication of a weak engine.
     given RPM.
2-11.           A03                                         2-11. Answer B. JSPT 2-27 (Part 43, Appendix D)
What is required by 14 CFR Part 43 appendix D when          According to 14 CFR 43, Appendix D, a cylinder com-
performing an annual/100-hour inspection on a recip-        pression check is required when performing an
rocating engine aircraft?                                   annual/100-hour inspection on a reciprocating engine
                                                            aircraft. Although a magneto timing check (answer A)
A    Magneto timing check.                                  and a valve clearance check (answer C) on
B    Cylinder compression check.                            nonhy-draulic Sifters are valuable checks that should be
C    Valve clearance check.                                 done periodically, they are not required on a 100-hour
                                                            inspection.
2-12.           A03                                         2-12. Answer C. JSPT 2-28 (ITP-P2)
After spark plugs from an opposed engine have been          When a spark occurs between the electrodes of a
serviced, in what position should they be reinstalled?      spark plug, metal is taken from one electrode and
                                                            deposited onto another. Therefore, when a spark plug
A    Next in firing order to the one from which they        fires positively, the ground electrode wears more than
     were removed.                                          the center electrode and when a spark plug fires nega-
B    Swapped bottom to top.                                 tively, the center electrode wears more than the ground
C    Next in firing order to the one from which they        electrode. Furthermore, lead and other impurities pro-
     were removed and swapped bottom to top.                duced during the combustion process tend to precipi-
                                                            tate to the lower spark plugs, causing them to wear. To
                                                            help equalize spark plug wear, each time spark plugs
                                                            are removed they should be replaced in the cylinder
                                                            next in the firing order to the one from which they were
                                                            removed and switched from top to bottom.
2-4                                                                                                    Turbine Engines
2-13.            A03                                        2-13. Answer C. JSPT 2-22 (AC 65-12A)
As the pressure is applied during a reciprocating engine    When a differential compression test is being performed
compression check using a differential pressure tester,     on an aircraft engine, the piston should be at top dead
what would a movement of the propeller in the               center when the air pressure is introduced into the cylin
direction of engine rotation indicate?                      der. If the piston is past top dead center, the air pressure
                                                            will force the piston to the bottom of the cylinder causing
A     The piston was on compression stroke.                 the propeller to rotate in the normal direction of rotation.
B     The piston was on exhaust stroke.                     Both answers (A) and (B) are incorrect because if the
C     The piston was positioned past top dead center.       piston was on the compression stroke the piston would
                                                            move counter to engine rotation and if the piston were on
                                                            the exhaust stroke the exhaust valve would be open and
                                                                                                         ;:
                                                            no pressure could be retained.            .:
2-14.           A03                                         2-14. Answer A. JSPT 2-22 (AC 65-12A)
Excessive valve clearance results in the valves opening     Excessive valve clearance describes a condition where
                                                            there is too much clearance between a rocker arm and
A     late and closing early.                               the end of a valve stem. The excessive clearance
B     early and closing late.                               results in the valve opening late due to the time
C     late and closing late.                                required for the rocker arm to contact the valve stem.
                                                            Furthermore, the valve will close early due to the
                                                            decreased dwell time.
2-15.            A03                                        2-15. Answer A. JSPT 2-32 (AC 65-12A)
During routine inspection of a reciprocating engine, a      Plain bearings used in aircraft engines are usually
deposit of small, bright, metallic particles which do not   made of nonferrous metals, such as silver, bronze, alu-
cling to the magnetic drain plug is discovered in the oil   minum, and various alloys of copper, tin, or lead. If this
sump and on the surface of the oil filter. This condition   type of material is found in the oil sump of an engine
                                                            and on the surface of the oil filter, it is an indication that
A     may be a result of abnormal plain type bearing        the bearings may be experiencing abnormal wear.
       wear and is cause for further investigation.         Answer (B) is incorrect because rings and cylinder
B     is probably a result of ring and cylinder wall wear   walls are made of ferrous metals that would cling to a
       and is cause for engine removal and/or overhaul.     magnetic drain plug, and answer (C) is wrong because
C     is normal in engines utilizing plain type bearings    the type of wear described is not normal.
       and aluminum pistons and is not cause for alarm.
2-16.            A03                                        2-16. Answer C. JSPT 2-38 (AC 65-12A)
A characteristic of dyna-focal engine mounts as             One characteristic of dyna-focal engine mounts is that
applied to aircraft reciprocating engines is that the       the shock mounts point toward the engine's center of
                                                            gravity. This design feature helps prevent vibration from
A     shock mounts eliminate the torsional flexing of       being transmitted to the airframe. The shock mounts
      the powerplant.                                       consist of a piece of rubber inside a round metal mount.
B     engine attaches to the shock mounts at the            The rubber within the mount helps absorb vibration and
      engine's center of gravity.                           allows some torsional flexing. Therefore, answer (A) is
C     shock mounts point toward the engine's center of      incorrect. Answer (B) is wrong because an engine's
      gravity.                                              center of gravity is inside the engine and you cannot
                                                            attach an engine mount inside the engine.
Turbine Engines                                                                                                     2-5
2-17.             A03                                         2-17. Answer B. JSPT 2-32 (AC 65-12A)
If metallic particles are found in the oil filter during an   Metal particles on engine oil screens or magnetic sump
inspection,                                                   plugs are generally an indication of partial internal
                                                              engine failure. However, due to the construction of air-
A    it is an indication of normal engine wear unless         craft oil systems, it is possible that metal particles could
      the particles are nonferrous.                           have collected in the oil system sludge at the time of a
B    the cause should be identified and corrected:            previous engine failure. At any rate, the cause or source
      before the aircraft is released for flight.             of the particles should be determined before the engine
C    it is an indication of normal engine wear unless         is returned to service. Answers (A) and (C) are
      the deposit exceeds a specified amount.                 incorrect because you cannot just assume the dis-
                                                              covery of metal particles is normal. The discovery of
                                                              metallic particles may indicate the beginning of a seri-
                                                              ous problem.
2-18.             A03                                         2-18. Answer A. JSPT 2-9 (AC 65-12A)
If the oil pressure gauge fluctuates over a wide range        The most likely cause of oil pressure fluctuating
from zero to normal operating pressure, the most likely       between zero and normal oil pressure is a low oil sup-
causeis.                                                      ply. If you have an engine with a low oil supply, the oil
                                                              pressure will be normal as long as the oil is being
A    low oil supply.                                          picked up by the pump. However, momentary losses of
B    broken or weak pressure relief valve spring.             oil pick-up will cause the oil pressure to drop to zero.
C    air lock in the scavenge pump intake.                    Answer (B) is incorrect because a weak or broken
                                                              pressure relief valve spring would result in low or pos-
                                                              sibly no oil pressure. Answer (C) is wrong because a
                                                              scavenge pump used in a dry-sump system pumps oil
                                                              from the engine sump back to the tank and has nothing
                                                              to do with the oil pressure within the system.
2-19.           A03                                           2-19. Answer A. JSPT 2-23 (AC 65-12A)
Which of the following is most likely to occur if an          When the valve clearance on a reciprocating engine is
overhead valve engine is operated with inadequate             inadequate, the push rods open the valves earlier and
valve clearances?                                             close them late. In addition, as the engine warms up,
                                                              the valve clearances tend to increase as the cylinders
A    The valve will not seat positively during start and      expand. Answer A is correct because the valves will not
     engine warmup.                                           remain seated as long during engine starting and
B    The further decrease in valve clearance that             warmup. Answer B is incorrect because the valve
     occurs as engine temperatures increase will cause        clearances increase as the engine warms up. Answer C
     damage to the valve-operating mechanism.                 is incorrect because the valves remain closed for a
C    The valves will remain closed for longer periods         shorter period with inadequate valve clearances.
     than specified by the engine manufacturer.
2-20.           A03                                           2-20. Answer B. JSPT 2-22 (AC 65-12A)
Excessive valve clearances will cause the duration of         When there is excessive clearance between the valve
valve opening to                                              stem and rocker arm (valve clearance), the valves will
                                                              not open as wide or remain open as long during engine
A increase for both intake and exhaust valves.                operation (answer B). This reduces the overlap period
B decrease for both intake and exhaust valves.                and the cylinder's volumetric efficiency.
C decrease for intake valves and increase for
exhaust valves.
2-6                                                                                                  Turbine Engines
2-21.                                                       2-21. Answer B. JSPT 2-32 (AC 65-12A) Engine
Engine crankshaft runout is usually checked                 crankshaft runout is typically checked when the
                                                            crankshaft is separated from the engine. Therefore,
1. during engine overhaul.                                  crankshaft runout is usually checked during an engine
2. during annual inspection.                                overhaul. Furthermore, manufacturers generally
3. after a "prop strike" or sudden engine stoppage.         require runout checks after sudden stoppage or a sud-
4. during 100-hour inspection.                              den reduction in speed, such as a prop strike. Answers
                                                            (A) and (C) are wrong because an engine is not disas-
A 1,3, and 4.                                               sembled down to the crankshaft during either an
B 1 and 3.                                                  annual or 100-hour inspection.
C 1,2 and 3.
2-22.           A03                                         2-22. Answer C. JSPT 2-13 (AC 65-12A)
Before attempting to start a radial engine that has been    Whenever a radial engine remains shut down for more
shut down for more than 30 minutes,                         than 30 minutes, oil and fuel may drain into the com-
                                                            bustion chambers of the lower cylinders or accumulate
A     turn the propeller by hand three or four revolu-      in the lower intake pipes. These fluids can cause a liq-
       tions in the opposite direction of normal rotation   uid lock, or hydraulic lock, which can damage the
       to check for liquid lock.                            engine if a start is attempted. To check for a liquid lock,
B     turn the ignition switch on before energizing the     the propeller should be turned by hand in the normal
       starter.                                             direction of rotation a minimum of two complete revolu-
C     turn the propeller by hand three to four              tions. Never attempt to clear a liquid lock by pulling the
       revolutions in the normal direction of rotation to   propeller through in the direction opposite the normal
       check for liquid lock.                               rotation (answer A), since this tends to inject the liquid
                                                            into the intake pipe, increasing the possibility of a lock
                                                            during a subsequent start. Answer (B) is incorrect
                                                            because turning the ignition switch on will not help
                                                            identify a liquid lock.
2-23.             A03                                       2-23. Answer A. JSPT 2-19 (AC 65-12A)
An engine misses in both the right and left positions of    The cold cylinder check determines the operating char-
the magneto switch. The quickest method for locating        acteristics of each cylinder of an air-cooled engine. The
the trouble is to                                           tendency for any cylinder or cylinders to be cold or to
                                                            be only slightly warm after the engine was running
A     check for one or more cold cylinders.                 indicates either a lack of combustion or incomplete
B     perform a compression check.                          combustion. If an engine misses in both the right and
C     check each spark plug.                                left positions of the magneto switch, combustion is not
                                                            taking place in one or more cylinders. Any time there is
                                                            a lack of combustion or incomplete combustion, the
                                                            cylinder(s) affected will feel cooler than the cylinder(s)
                                                            where complete combustion is occurring. Answer (B) is
                                                            incorrect because conducting a compression check will
                                                            not identify the cause of a misfire and answer (C) is
                                                            wrong because checking each spark plug is not the
                                                            quickest way to locate a misfiring cylinder.
2-24.            A03                                        2-24. Answer B. JSPT 2-22 (AC 65-12A) Exhaust
A hissing sound from the exhaust stacks when the pro-       valve blow-by occurs when the exhaust valve does not
peller is being pulled through manually indicates           seat properly, allowing a portion of the fuel/air charge to
                                                            escape before combustion takes place. Exhaust valve
A     a cracked exhaust stack.                              blow-by is identified by a hissing or whistling sound
B     exhaust valve blow-by.                                coming from the exhaust stacks. Answer (A) is
C     worn piston rings.                                    incorrect because you could never get enough air to
                                                            pass through the exhaust stack to produce a hissing
                                                            sound in a cracked exhaust. Answer (C) is wrong
                                                            because worn piston rings produce more of a grinding
                                                            sound than a hissing sound.
Turbine Engines                                                                                                    2-7
2-25.                                                          2-25. Answer B. JSPT 2-22 (AC 65-12A)
If an engine cylinder is to be removed, at what position       Before removing a cylinder from an engine, the piston
in the cylinder should the piston be?                          should be at top dead center on the compression
                                                               stroke (answer B). Having the piston in this position
A    Bottom dead center.                                       helps prevent damage to the cylinder, piston, and
B    Top dead center.                                          valves and helps relieve pressure on both the intake
C    Halfway between top and bottom dead center.               and exhaust rocker arms.
2-26.           A03                                            2-26. Answer C. JSPT 2-33 (AC 65-12A)
Engine operating flexibility is the ability of the engine to   Operating flexibility is defined as the ability of an
                                                               engine to run smoothly and give desired performance
A    deliver maximum horsepower at a specific altitude.        at all engine speeds. Both answers (A) and (B) are
B    meet exacting requirements of efficiency and low          wrong because delivering maximum horsepower at a
      weight per horsepower ratio.                             specific altitude and meeting requirements of efficiency
C    run smoothly and give the desired performance at          and weight to horsepower ratio are not part of operat-
      all speeds.                                              ing flexibility.
2-27.            A03                                           2-27. Answer B. JSPT 2-60 (AC 65-12A)
Standard aircraft cylinder oversizes usually range from        Generally, standard aircraft cylinder oversizes are
0.010 inch to 0.030 inch. Oversize on automobile               0.010 inch, 0.015 inch, 0.020 inch, or 0.030 inch. The
engine cylinders may range up to 0.100 inch. This is           reason aircraft cylinders cannot be oversized as much
because aircraft engine cylinders                              as automobile cylinders is because aircraft cylinders
                                                               have relatively thin walls and may have a nitrided sur-
A    have more limited cooling capacity.                       face. Both answers (A) and (C) are incorrect because
B    have relatively thin walls and may be nitrided.           aircraft cylinders do not have a more limited cooling
C    operate at high temperatures.                             capacity nor do they operate at excessively high tem-
                                                               peratures.
2-28. Reserved                                                 2-28. Reserved
2-29.           A03                                            2-29. Answer C. JSPT 2-19 (AC 65-12A)
During ground check an engine is found to be                   The most likely cause of a rough running engine which
rough-running, the magneto drop is normal, and the             has normal magneto drop and high manifold pressure
manifold pressure is higher than normal for any given          is a dead cylinder (answer C). Answer (A) is incorrect
RPM. The trouble may be caused by                              because if several spark plugs were fouled on different
                                                               cylinders the magneto drop would not be normal.
A    several spark plugs fouled on different cylinders.        Answer (B) is wrong because, although a leak in an
B    a leak in the intake manifold.                            intake manifold could cause a rough running engine,
C    a dead cylinder.                                          manifold pressure would not necessarily be higher
                                                               than normal at all rpm settings.
2-30.            A03                                           2-30. Answer A. JSPT 2-28 (AC 65-12A)
What is the best indication of worn valve guides?              If the valve guides of an engine are worn, there will be
                                                               excessive clearance between the valve guide and the
A    High oil consumption.                                     valve stem. The excessive clearance allows oil to seep
B    Low compression.                                          by the valve stems and enter the intake and exhaust
C    Low oil pressure.                                         ports, causing high oil consumption. Answer (B) is
                                                               wrong because, as long as the valves are seating prop-
                                                               erly, worn valve guides will not affect cylinder compres-
                                                               sion. Answer (C) is incorrect because the oil that seeps
                                                               by the valve stems is not under pressure and, therefore,
                                                               cannot in itself cause a low oil pressure reading.
2-8                                                                                                 Turbine Engines
2-31.            A04                                       2-31. Answer A. JSPT 2-66 (AC 65-12A)
Direct mechanical push-pull carburetor heat control        When rigging any carburetor control linkage, the compo-
linkages should normally be adjusted so that the stop      nent being moved must contact its stop prior to the stop
located on the diverter valve will be contacted            in the cockpit is reached. This ensures full control travel.
                                                           Answers (B) and (C) are incorrect because they both
A     before the stop at the control lever is reached in   indicate conditions where the cabin control reaches its
      both HOT and COLD positions.                         stop prior to the component reaching its stop.
B     before the stop at the control lever is reached in
      the HOT position and after the stop at the control
       lever is reached in the COLD position.
C     after the stop at the control lever is reached in
      both HOT and COLD positions.
2-32.                                                      2-32. Answer B. JSPT 2-66 (AC 65-12A)
Which of the following engine servicing operations         After an engine is installed it should be pre-oiled prior to
generally requires engine pre-oiling prior to starting     starting. Pre-oiling helps prevent excessive wear or
the engine?                                                failure of the engine bearings. Both answers (A) and (C)
                                                           are incorrect because neither operation depletes the
A     Engine oil and filter change.                        supply of residual oil within an engine.
B     Engine installation.
C     Replacement of oil lines.
2-33.           A04                                        2-33. Answer C. JSPT 2-67 (AC 65-12A)
During the inspection of an engine control system in       After a push/pull control rod has been adjusted you
which push-pull control rods are used, the threaded rod    should check the number of threads engaging the rod
ends should                                                end. To check for the proper amount of engagement, an
                                                           inspection hole is typically provided in which a piece of
A     not be adjusted in length for rigging purposes       safety wire is inserted. If the safety wire can pass
      because the rod ends have been properly posi-        through the hole, there is insufficient thread engagement.
      tioned and staked during manufacture.                Answer (A) is incorrect because push-pull rods must be
B     be checked for thread engagement of at least two     adjusted from time to time. Answer (B) is wrong because
      threads but not more than four threads.              the minimum number of threads engaged is typically
C     be checked for the amount of thread engagement       greater than two threads and there is no maximum
      by means of the inspection holes.                    thread engagement requirement for rod ends.
2-34.           A04                                        2-34. Answer C. JSPT 2-66 (AC 65-12A)
Which of the following would most likely cause a           An extremely lean mixture will either not burn at all or
reciprocating engine to backfire through the induction     burn so slowly that combustion continues until the
system at low RPM operation?                               intake valve opens near the end of the exhaust stroke.
                                                           When this happens, the flame in the cylinder ignites
A     Idle mixture too rich.                               the contents in the intake manifold causing an explo-
B     Clogged derichment valve.                            sion, or backfire within the induction manifold. Answer
C     Lean mixture.                                        (A) is wrong because an excessively rich mixture
                                                           causes flooding and spark plug fouling, not backfiring.
                                                           Answer (B) is incorrect because there is no such thing
                                                           as a derichment valve.
2-35.          A04                                         2-35. Answer B. JSPT 2-66 (AC 65-12A)
How may it be determined that a reciprocating engine       When an engine is being pre-oiled, a line from the inlet
with a dry sump is pre-oiled sufficiently?                 side of the engine-driven oil pump must be disconnected
                                                           to permit the pre-oiler tank to be connected. Then, a line
A     The engine oil pressure gauge will indicate normal   near the nose of the engine is disconnected to allow oil to
      oil pressure.                                        flow out. Once oil flows out of the engine pre-oiling is
B     Oil will flow from the engine return line or         complete. Answer (A) is incorrect because the oil pres-
      indicator port.                                      sure gauge will not indicate an oil pressure when pre-oil-
C     When the quantity of oil specified by the manu-      ing, and answer (C) is wrong because manufacturers do
      facturer has been pumped into the engine.            not specify the specific oil quantity used to pre-oil.
Turbine Engines                                                                                                       2-9
2-36.           A04                                               2-36. Answer A. JSPT 2-19 (AC 65-12A)
Backfiring through the carburetor generally results               An extremely lean mixture will either not bum at all or
from the use of                                                   burn so slowly that combustion continues until the
                                                                  intake valve opens near the end of the exhaust stroke.
A   an excessively lean mixture.                                  When this happens, the flame in the cylinder ignites the
B   excessively atomized fuel.                                    contents in the intake manifold causing an explosion
C   an excessively rich mixture.                                  known as a backfire. Answer (B) is wrong because
                                                                  excessively atomized fuel typically burns completely
                                                                  and rapidly making a backfire impractical, and answer
                                                                  (C) is incorrect because an excessively rich mixture
                                                                  causes flooding and spark plug fouling, not backfiring.
                                                                  2-37. Answer C. JSPT 2-22 (AC 65-12A)
2-37.              A04                                            Excessive leakage past the piston rings can be
If air is heard coming from the crankcase breather or             detected by the sound of escaping air at the engine
oil filler during a differential compression check, what          breather tube or oil filler cap. Leakage at the exhaust
is this an indication of?                                         valve (answer A) is detected by listening for air at the
                                                                  exhaust outlet, and intake valve leakage (answer B) is
A   Exhaust valve leakage.                                        detected at the air intake.
B   Intake valve leakage.
C   Piston ring leakage.
2-38.           A04                                               2-38. Answer C. JSPT 2-19 (AC 65-12A)
One cause of afterfiring in an aircraft engine is                 Afterfiring refers to a condition when unburned fuel
                                                                  from an excessively rich fuel/air mixture combines with
A   sticking intake valves.                                       air in the exhaust stacks and ignites, or fires, in the
B   an excessively lean mixture.                                  exhaust system. Sticking intake valves (answer A)
C   an excessively rich mixture.                                  cause timing conflicts that can lead to flash back in the
                                                                  intake pipe and high engine temperatures, whereas an
                                                                  excessively lean mixture (answer B) causes backfiring.
2-39.           B02                                               2-39. Answer C. JSGT 2-22 (AC 65-12A)
Standard sea level pressure is                                    A standard day is defined by a sea level pressure of
                                                                  29.92 inches of mercury, or 14.7 psi. Both answers (A)
A 29.00" Hg.                                                      and (B) are wrong because they are less than 29.92
B 29.29" Hg.                                                      inches.
C 29.92" Hg.
2-40.           B02                                               2-40. Answer A. JSGT 2-22 (AC 65-12A)
Using standard atmospheric conditions, the standard               A standard day is defined by an atmospheric pressure
sea level temperature is                                          of 29.92 inches of mercury, or 14.7 psi, and a
                                                                  temperature of 59 F or 15 C.
A     59  F.
B    59  .C
C    29 C.
2-41.             B03                                             2-41. Answer C. JSGT 2-16 (AC 65-9A) Newton's First
Newton's First Law of Motion, generally termed the                Law of Motion states that any body at rest will remain at
Law of Inertia, states                                            rest and any body in motion will remain in a straight line
                                                                  motion, unless acted upon by some outside force.
A   To every action there is an equal and opposite reaction.      Answer (A) is Newton's Third Law of Motion and
B   Force is proportional to the product of mass and              answer (B) is a statement from Newton's Second Law.
     acceleration.
C   Every body persists in its state of rest, or of motion in a
     straight line, unless acted upon by some outside force.
2-42. Reserved                                                    2-42. Reserved
2-10                                                                                              Turbine Engines
2-43.            C01                                     2-43. Answer A. JSGT 14-3 (AC39-7B)
(Refer to figure 1) Determine which portion of the AD    Paragraph B in the AD applies to all model 0-690
is applicable for Model 0-690 series engine, serial      engines with serial numbers 5265-40 to 6129-40. The
No.5863-40 with 283 hours time in service.          ;;   engine and serial number listed in the question fall
A (B),(l).                                               within this listing. Paragraph 1 also applies to the listed
B (A).                                                   engine because it identifies engines with more than
C (B),(2).                                               275 hours time in service. Answer (B) is wrong
                                                         because paragraph A of the AD applies only to serial
                                                         numbers 101-40 through 5264-40 for the model O-690
                                                         engine and the engine in this question falls outside this
                                                         range. Answer (C) is wrong because subparagraph 2
                                                         applies to engines with less than 275 hours time in ser-
                                                         vice, and the engine in the example has 283 hours.
2-44.            C01
A Cessna 180 aircraft has a McCauley propeller Model     2-44. Answer B. JSGT 14-8 (AC 65-9A)
No.2A34C50/90A. The propeller is severely damaged        The Aircraft Specifications or Type Certificate Data
in a ground accident, and this model propeller is not    Sheet for an aircraft lists the engines and propellers
available for replacement. Which of the following        approved for use on the aircraft. If there is more than
should be used to find an approved alternate replace-    one approved propeller for the Cessna 180, it will be
ment?                                                    listed in one of these documents. This information can
                                                         also be found in the Summary of Supplemental Type
A   Summary of Supplemental Type Certificates.           Certificates. Therefore, there are two correct answers
B   Aircraft Specifications/Type Certificate Data        for this question. However, it is our belief that the
    Sheets.                                       :      Aircraft Specifications or Type Certificate Data Sheets
C   Aircraft Engine and Propeller Specifications/        are better references to use. Answer (C) is incorrect
    Type Certificate Data Sheets.                        because the Engine and Propeller Specifications/Type
                                                         Certificate Data Sheets do not state which propellers
                                                         may be installed on a given aircraft.
2-45.            C01
(1) Airworthiness Directives are Federal Aviation        2-45. Answer C. JSGT 14-3 (AC 65-9A)
    Regulations and must be complied with unless         Both statements (1) and (2) are correct. Airworthiness
    specific exemption is granted.                       Directives (ADs) are part of 14 CFR Part 39 and must
(2) Airworthiness Directives of an emergency             be complied with unless a specific exemption is
    nature require immediate compliance upon receipt.    granted. Statement (2) is also true on most occasions,
                                                         because emergency ADs generally do require immedi-
Regarding the above statements,                          ate compliance. Furthermore, if an AD is issued that
                                                         identifies an emergency condition, compliance is typi-
A   only No. 1 is true.                                  cally required upon receipt.
B   only No. 2 is true.
C   both No. 1 and No. 2 arc true.
2-46.           C01
Which of the following contains a minimum checklist      2-46. Answer B. JSGT 2-26 (14 CFR Part 43,
for 100-hour inspections of engines?                     Appendix D)
                                                         14 CFR Part 43 contains the minimum checklist for a
A 14 CFR Part 33, Appendix A.                            100-hour inspection of an engine and airframe. Answers
B 14 CFR Part 43, Appendix D.                            (A) and (C) are incorrect because neither Appendix A of
C Engine Specifications or Type Certificate Data         14 CFR Part 33, nor the Type Certificate Data Sheets
Sheets.                                                  contain information on inspection checklists.
Turbine Engines                                                                                                2-11
    This is the compliance portion of an FAA Airworthiness Directive.                          >
    Compliance required as indicated:
    (A) For model O-690 series engines, serial Nos. 101-40 through 5264-40 and IO-690
    series engines, serial Nos. 101-48 through 423-48, compliance with (C) required within
    25 hours' time in service after the effective date of this AD and every 100 hours' time in
    service thereafter.
    (B) For model O-690 series engines, serial Nos. 5265-40 through 6129-40 and IO-690
    series engines, serial Nos. 424-48 through 551-48, compliance with (C) required as
    follows:                                ,
      (1) Within 25 hours' time in service after the effective date of this AD and every 100
    hours' time in service thereafter for engines with more than 275 hours' time in service
    on the effective date of this AD.
      (2) Prior to the accumulation of 300 hours total time in service and every 100 hours'
    time in service thereafter for engines with 275 hours or less time in service on the
    effective date of this AD.
    (C) Inspect the oil pump drive shaft (P/N 67512) on applicable engines in accordance
    with instructions contained in Connin Service Bulletin No. 295. Any shafts which are
    found to be damaged shall be replaced before further flight. These inspections shall
    be continued until Connin P/N 67512 (redesigned) or P/N 74641 oil pump drive shaft
    is installed at which time the inspections may be discontinued.
                                     Figure 1. Airworthiness Directive Excerpt.
2-47.
Which of the following contains a table that lists the        2-47. Answer B. JSGT 14-8 (AC 65-12A)
engines to which a given propeller is adaptable?              To find out what engines a particular propeller is
                                                              adaptable to, you must look at the propeller's Type
A    Aircraft Type Certificate Data Sheets.                   Certificate Data Sheet. Answer (A) is incorrect because
B    Propeller Type Certificate Data Sheets.                  Aircraft Type Certificate Data Sheets give the propeller
C    Engine Type Certificate Data Sheets.                     that is acceptable for use on a given aircraft, and
                                                              answer (C) is wrong because Engine Type Certificate
                                                              Data Sheets list the propellers that are acceptable for
                                                              use on a given engine.
2-48.           C01
Which of the following component inspections is to be         2-48. Answer B. JSGT 2-27 (14 CFR Part 43,
accomplished on a 100-hour inspection?                        Appendix D)
                                                              According to Appendix D of 14 CFR Part 43 a 100-hour
A    Check internal timing of magneto.                        inspection on an engine requires that a cylinder com-
B    Check cylinder compression.                              pression check be performed. Although it is good prac-
C    Check valve timing.                                      tice to also check the internal timing of the magnetos
                                                              (answer A) and the valve timing (answer C), neither is
                                                              required to be checked during a 100-hour inspection.
2-12                                                                                                  Turbine Engines
2-49.          C01                                        2-49. Answer B. JSGT 14-8
Where would one find type design information for an       Both the R1830-92 and DC-3 were certified under the
Rl830-92 engine certificated under the Civil Air          older Civil Aeronautics Regulations and, therefore, Type
Regulations (CAR) and installed on a DC-3?                Certificate Data Sheets were not published for either the
                                                          engine or the aircraft. Therefore, the only place to find type
A    The Aircraft Specifications and Type Certificate     design information for an R1830-92 engine that is installed
     Datasheet.                                           on a DC-3 is in the aircraft engine specifications. Answer (A)
B    The Aircraft Engine Specifications.                  is incorrect because the Aircraft Specifications and Type
C    The Aircraft Engine Type Certificate Handbook.       Certificate Data Sheets do not contain engine type design
                                                          information and answer (C) is incorrect because there are
                                                          no Type Certificate Handbooks.             i
2-50.            C01                                      2-50. Answer A. JSPT 2-42 (AC 65-12A)
The breaking loose of small pieces of metal from          Flaking is defined as the breaking loose of small pieces
coated surfaces, usually caused by defective plating or   of metal from coated surfaces. It is usually caused by
excessive loads, is called                                defective plating or excessive loading. Answer (B) is
                                                          wrong because chafing is wear caused by light rubbing
A    flaking.                                             between two parts that does not produce small pieces
B    chafing.                                             of metal. Answer (C) is incorrect because brinelling
C    brinelling.                                          refers to the indentations found on bearing races that
                                                          are caused by high static loads or the application of
                                                          force during component installation or removal.
2-51.          C01                                        2-51. Answer A. JSGT 14-8 (14 CFR 23.903)
Each powerplant installed on an airplane with a           Each airplane having a standard airworthiness certificate
Standard Airworthiness Certificate must have been         must be equipped with an engine which is type certificated
                                                          and, if it is prop driven, a type certificated propeller. Answer
A    type certificated.                                   (B) is incorrect because an engine that is type certificated
B    manufactured under the TSO system.                   does not need to be manufactured under a TSO, and
C    originally certificated for that aircraft.           answer (C) is wrong since Supplemental Type Certificates
                                                          permit the installation of components and appliances that
                                                          were not originally certificated for a given aircraft.
2-52.            C01                                      2-52. Answer C. JSPT 2-42 (AC 65-12A) Galling is
A severe condition of chafing or fretting in which a      defined as a severe condition of chafing or fretting in
transfer of metal from one part to another occurs is      which a transfer of metal from one part to another
called                                                    occurs. It is usually caused by a slight movement of
                                                          mated parts under high loads and having limited relative
A    scoring.                                             motion. Answer (A) is incorrect because scoring results
B    burning.                                             in deep scratching and is the result of foreign particles
C    galling.                                             between moving parts. Answer (B) is wrong because
                                                          burning is surface damage caused by excessive heat
                                                          and rarely results in the transfer of metal.
2-53.           C01                                       2-53. Answer B. JSPT 2-42 (AC 65-12A)
Indentations on bearing races caused by high static       Brinelling is defined as one or more indentations on
loads are known as                                        bearing races, usually caused by high loads or the
                                                          application of force during installation or removal. The
A   fretting.                                             indentations are rounded or spherical due to the
B   brinelling.                                           impression left by the contacting ball or roller bearings.
C    galling.                                             Answer (A) is incorrect because fretting is a form of
                                                          surface corrosion that is caused by movement between
                                                          two parts that are clamped together under pressure.
                                                          Answer (C) is wrong because galling is a severe
                                                          chafing that results in transfer of metal.
Turbine Engines                                                                                               2-13
2-54.          C01                                        2-54. Answer C. JSGT 14-8 (AC 65-9A)
When inspecting an aircraft reciprocating engine what     When inspecting an engine, it is important to make
document is used to determine if the proper magnetos      sure that it conforms to its original type design. The
are installed?                                            original type design for an engine manufactured before
                                                          1959 is contained in the Aircraft Engine Specifications.
A    Instructions for continued airworthiness issued by   However, for engines that were manufactured after
     the engine manufacturer.                             1959 the type design information is in the Type
B    Engine Manufacturer's Maintenance Manual.            Certificate Data Sheet. Answer (A) is wrong because
C    Aircraft Engine Specifications or Type Certificate   instructions for continued airworthiness consist of
     Data Sheets.                                         maintenance manuals, installation instructions, and
                                                          servicing information. Answer (B) is wrong because
                                                          maintenance manuals do not reference approved
                                                          components for a given engine or aircraft.
2-55.           C01                                       2-55. Answer C. JSGT 14-4 (14 CFR 65.95)
Which of the following can inspect and approve an         Although a certified airframe and powerplant technician
engine major repair for return to service?                is authorized to perform a major repair, it takes a person
                                                          with the inspection authorization to return the repair to
A    Certificated mechanic with airframe and              service. Answers (A) and (B) are incorrect because a
     power-plant ratings.                                 mechanic certificate alone is not sufficient to inspect
B    Certificated mechanic with a powerplant rating.      and approve a major repair for return to service.
C    Certificated mechanic with inspection authoriza-
     tion.
2-56.          C01                                        2-56. Answer B. JSGT 14-4 (14 CFR Part 43,
What publication is used for guidance to determine        Appendix A)
whether a powerplant repair is major or minor?            A list of what constitutes a powerplant major repair or
                                                          alteration is provided in FAR Part 43, Appendix A.
A irworthiness Directives.                                However, this list is only a guide and does not identify
B Federal Aviation Regulations, Part 43, appendix A.      all possible repairs and alterations. Answer (A) is
C Technical Standard Orders.                              incorrect because Airworthiness Directives give no
                                                          guidance on the status of repairs and alterations and
                                                          answer (C) is incorrect because a technical standard
                                                          order merely certifies that a part meets certain quality
                                                          standards.
2-57.             C01                                     2-57. Answer B. JSGT 14-8 (FAR 23.1)
The airworthiness standards for the issue of type         FAR Part 23 entitled, Airworthiness Standards: Normal,
certificates for small airplanes with nine or less        Utility, Acrobatic and Commuter Category Airplanes,
passenger seats in the normal, utility, and acrobatic     prescribes the airworthiness standards for the issue of
categories may be found in the                            type certificates for small airplanes in the normal, util-
                                                          ity, and acrobatic categories that have a passenger
A    Supplemental Type Certificate.                       seating configuration, excluding pilot seats, of 9 seats
B    Federal Aviation Regulations, Part 23.               or less. Answer (A) is incorrect because a
C    Federal Aviation Regulations, Part 21.               Supplemental Type Certificate allows the alteration of
                                                          an aircraft, powerplant, or component and contains no
                                                          airworthiness information beyond the requirements for
                                                          the alteration. Answer (C) is wrong because Part 21 of
                                                          the FARs gives the certification procedures necessary
                                                          to obtain a Type Certificate but does not provide the
                                                          required standards.
2-14                                                                                             Turbine Engines
2-58.           C01                                       2-58. Answer C. JSGT 14-4 (AC 65-19E)
Which of the following contains approved data for         Manufacturer's maintenance instructions are accept-
performing a major repair to an aircraft engine?          able to use when performing a major repair to an
                                                          engine, providing they are FAA approved. FAA
A Engine Type Certificate Data Sheets.                    approval must be stamped on the manual before it can
B Supplemental Type Certificates.                         be used as approved data. Answer (A) is incorrect
C Manufacturer's maintenance instructions when            because Engine Type Certificate Data Sheets contain
FAA approved.                                             information such as power rating, fuel grade, and
                                                          weight for a given engine model but contain no
                                                          approved data for major repairs. Answer (B) is incorrect
                                                          because Supplemental Type Certificates contain
                                                          approved data for major alterations, not repairs.
2-59.            C01                                      2-59. Answer B. JSGT 14-22 (FAR 43.9)
What maintenance record(s) is/are required following a    When a major repair is performed on an engine, an
major repair of an aircraft engine?                       entry must be made in the engine's maintenance
                                                          records and an FAA Form 337 must be filled out. One
A   Entries in engine maintenance records and a list      copy of the Form 337 stays with the maintenance
    of discrepancies for the FAA.                         records and a second copy is sent to the FAA. An
B   Entries in the engine maintenance record and          exception to this would be if the repair was done by a
    FAAForm337.                                           certified repair station. If approved data was used, the
C   Entry in logbook.                                     repair station would not be required to fill out a Form
                                                          337. Answer (A) is wrong because there is no
                                                          requirement that the FAA receive a list of discrepancies
                                                          found during a major repair, and answer (C) is incorrect
                                                          because an FAA Form 337 must be completed in
                                                          addition to an entry in the engine's maintenance record.
2-60.           C01                                       2-60. Answer A. JSPT 2-32 (AC 43.13-1B)
A ground incident that results in propeller sudden        When an engine has been subjected to a sudden
stoppage may require a crankshaft runout inspection.      stoppage requiring a crankshaft run-out check, it
What publication would be used to obtain crankshaft       should be done in accordance with the manufacturer's
runout tolerance?                                         technical data. Therefore, the manufacturer's mainte-
                                                          nance instructions would typically be used. Answers (B)
A Current Manufacturer's maintenance instructions.        and (C) are incorrect because neither the Type
B Type Certificate Data Sheet.                            Certificate Data Sheets nor AC 43.13-1B gives specific
C AC 43.13-1 A, Acceptable Methods, Techniques,           limits or tolerances for engine overhauls.
and Practices Aircraft Inspection and Repair.
2-61.                                                     2-61. Answer A. JSGT 14-3 (AC 65-9A)
Select the Airworthiness Directive applicability          Answer (A) applies to the engine identified because it
statement which applies to an IVO-355 engine, serial      gives the model number as IVO-355; the serial num-
number T8164, with 2,100 hours total time and 300         bers include the number of the listed engine; and the
hours since rebuilding.                                   hours of time in service apply to the listed engine. The
                                                          number of hours the engine has accumulated since
A   Applies to all IVO-355 engines, serial numbers        rebuilding does not matter in relation to this question.
    T8000 through T83OO, having less than 2,400           Answer (B) is incorrect since it specifies engines with
    hours total time.                                     more total time than the given engine has, and answer
B   Applies to all IVO-355 engines, serial numbers        (C) is wrong because it specifies IO and TVIO-355
    T8000 through T8900 with 2,400 hours or more          engines while the given engine is an IVO-355.
    total time.
C   Applies to all I.O. and TV10-355 engines, all ser-
    ial numbers regardless of total time or since over-
    haul.
Turbine Engines                                                                                                   2-15
2-62.                                                         2-62. Answer C. JSGT 14-12 (Part 33, Appendix A)
What section in the instructions for continued airwor-        According to FAR Part 33 Appendix A, the Instructions
thiness is FAA approved?                                      for Continued Airworthiness must contain a section
                                                              titled Airworthiness Limitations that is segregated and
A    Engine maintenance manual or section.                    clearly distinguishable from the rest of the document.
B    Engine overhaul manual or section.                       This section is FAA approved and must list the mandatory
C    Airworthiness limitations section.                       replacement time, inspection interval, and related
                                                              procedures required for certification. Answers (A) and
                                                              (B) are incorrect because the FARs do not require
                                                              engine maintenance and overhaul manuals to be FAA
                                                              approved.
2-63.            HOI                                          2-63. Answer A. JSGT 2-5 (AC 65-12A)
Which unit most accurately indicates fuel consumption         Electronic fuel flowmeters (vane-type or mass-flow type)
of a reciprocating engine?                                    are what many modern aircraft use to measure the
                                                              amount of fuel consumed by an engine. Fuel
A    Fuel flowmeter.                                          flowmeters monitor the amount of fuel that flows past a
B    Fuel pressure gauge.                                     given point and display this flow rate in the cockpit as
C    Electronic fuel quantity indicator.                      pounds of fuel consumed per hour. Answer (B) is
                                                              wrong because a fuel pressure gauge only indicates
                                                              the fuel pressure within the fuel lines and answer (C) is
                                                              incorrect because an electronic fuel quantity indicator
                                                              displays the amount of fuel in the fuel tanks and does
                                                              not directly indicate the amount of fuel consumption.
2-64.             HOI                                         2-64. Answer B. JSPT 2-5 (ITP-A2)
The fuel flowmeter used with a continuous-fuel injection      Most light aircraft equipped with continuous-fuel injec-
system installed on an aircraft horizontally opposed recip-   tion systems utilize a fuel flow indication system that
rocating engine measures the fuel pressure drop across the    measures the pressure drop across the injection noz-
                                                              zles to determine fuel flow (answer B). With this type of
A    manifold valve.                                          system, a higher fuel flow results in a greater pressure
B    fuel nozzles.                                            drop and a corresponding increase in fuel flow is indi-
C    metering valve.                                          cated in the cockpit. Answer (A) and (C) are wrong
                                                              because there is no pressure drop across either the
                                                              manifold valve or metering valve.
2-65.             HOI                                         2-65. Answer C. JSPT 2-5 (ITP-A2)
The principle fault in the pressure type fuel flowmeter       Most light aircraft equipped with continuous-fuel injec-
indicating system, installed on a horizontally opposed        tion systems utilize a fuel flow indication system that
continuous-flow fuel injected aircraft reciprocating          measures the pressure drop across the injection noz-
engine, is that a plugged fuel injection nozzle will          zles to determine fuel flow. With this type of system, a
cause a                                                       higher fuel flow results in a greater pressure drop and a
                                                              corresponding increase in fuel flow is indicated in the
A    normal operation indication.                             cockpit. However, if an injector nozzle becomes
B    lower than normal fuel flow indication.                  restricted, the pressure drop across the nozzle
C    higher than normal fuel flow indication.                 becomes greater and produces a false or high fuel flow
                                                              reading (answer C).
2-66.            HOI                                          2-66. Answer A. JSPT 2-6 (AC 65-15A)
Motor driven impeller and turbine fuel flow transmit-         In an autosyn system installed in the fuel system of tur-
ters are designed to transmit data                            bine engine aircraft, fuel-flow data are transmitted using
                                                              the aircraft's electrical system. Answers (B) and (C) are
A    using aircraft electrical system power.                  incorrect because mechanical connections or fuel lines
B    mechanically.                                            between an engine and a cockpit indicator would be
C    by fuel pressure.                                        inaccurate, hazardous, and add unnecessary weight.
2-16                                                                                               Turbine Engines
2-67.           HOI                                        2-67. Answer A. JSPT 2-6 (AC 65-15A)
The fuel-flow indicator rotor and needle for a             The only fuel flow indicating system that utilizes an
motor-impeller and turbine indicating system is driven     impeller and turbine is the synchronous mass flow sys-
by                                                         tem. In this type of system both the indicator rotor and
                                                           needle are driven by an electrical signal. Answers (B)
A      an electrical signal.                               and (C) are incorrect because there is no fuel flow indi-
B      direct coupling to the motor shaft.                 cating system that is driven directly off a motor shaft or
C      a mechanical gear train.                            through a mechanical gear train.
2-68.            HOI                                       2-68. Answer B. JSPT 2-5 (ITP-A2)
On a twin-engine aircraft with fuel-injected reciprocat-   Most light twin-engine aircraft utilize a fuel flow indica-
ing engines, one fuel-flow indicator reads considerably    tion system that measures the pressure drop across the
higher than the other in all engine operating              injection nozzles to determine fuel flow. With this type of
configurations. What is the probable cause of this         system, a higher fuel flow results in a greater pressure
indication?                                                drop and a corresponding increase in fuel flow is
                                                           indicated in the cockpit. However, if an injector nozzle
A      Carburetor icing.                                   becomes restricted, the pressure drop across the nozzle
B      One or more fuel nozzles are clogged.               becomes greater and produces a false or high fuel flow
C      Alternate air door stuck open.                      reading. Answer (A) is incorrect because you can not
                                                           get carburetor icing with a fuel-injected engine, and
                                                           answer (C) is wrong because, although having an alter-
                                                           nate air door stick open will cause a decrease in power
                                                           output, it will have little effect on fuel flow indications.
                                                           2-69. Answer B. JSPT 2-5 (AC 65-15A)
2-69.            HOI                                       The fuel flow indication used for fuel injected opposed
The fuel-flow indication system used with many             engines is actually a measure of the pressure drop
fuel-injected opposed engine airplanes utilizes a          across the fuel injection nozzles. With this type of sys-
measure of                                                 tem, a higher fuel flow creates a greater pressure drop
                                                           and a corresponding increase in the indicated fuel flow.
A       fuel flow volume.                                  Answer (A) is incorrect because volume measuring
B      fuel pressure.                                      systems (autosyn or vane) are used on large recipro-
C     fuel flow mass.                                      cating engine aircraft and answer (C) is wrong because
                                                           mass measuring systems are typically used on turbine
                                                           engine aircraft.
                                                           2-70. Answer B. JSPT 2-6 (ITP-P2)
                                                           A computerized fuel system (CFS) utilizes a transducer
2-70.            HOI                                       mounted in the fuel line leading to the engine to provide
In addition to fuel quantity, a computerized fuel system   fuel flow in gallons or pounds per hour, gallons or
(CFS) with a totalizer-indicator provides indication of    pounds remaining, time remaining for flight at the
how many of the following?                                 current power setting, and gallons used since startup
                                                           or reset. Of the four items listed, three are provided by
1.   Fuel flow rate.                                       a CFS.
2.   Fuel used since reset or initial start-up.
3.   Fuel time remaining at current power setting.
4.   Fuel temperature.
A      TWO.
B      Three.
C      Four.
Turbine Engines                                                                                                  2-17
2-71.             HOI                                       2-71. Answer A. JSPT 2-6 (AC 65-9A)
The fuel-flow indication data sent from motor driven        In both a motor driven impeller and turbine, and a
impeller and turbine, and motorless type fuel flow          motorless fuel flow indication system a flow meter
transmitters is a measure of                                transmitter converts the fuel's mass-flow rate into elec-
                                                            tronic signals that produce a fuel flow indication in the
A    fuel mass-flow.                                        cockpit. Answer (B) is incorrect because fuel
B    fuel volume-flow.                                      volume-flow transmitters typically utilize a movable
C    engine burner pressure drop.                           vane, and answer (C) is incorrect because engine
                                                            burner can pressure is not a monitored engine
                                                            parameter and, in any case, is only indirectly related to
                                                            engine fuel flow.
2-72.
In an aircraft equipped with a pressure-drop type           2-72. Answer B. JSPT 2-5 (ITP-A2)
fuel-flow indicating system, if one of the injector         If an injector nozzle becomes restricted in an aircraft
nozzles becomes restricted, this would cause a decrease     equipped with a pressure-drop type fuel flow indicating
in fuel flow with                                           system the pressure drop across the nozzle increases
                                                            and produces a false or high fuel flow indication. Both
A    a decreased fuel flow indication on the gauge.         answers (A) and (C) are wrong because a restricted
B    an increased fuel flow indication on the gauge.        injector nozzle will not cause a decrease or no change
C    no change in fuel flow indication on the gauge.        in the fuel flow indication.
2-73.          H02
A manifold pressure gauge is designed to                    2-73. Answer C. JSPT 2-6 (AC 65-12A)
                                                            A manifold pressure gauge measures the absolute
A    maintain constant pressure in the intake manifold.     pressure in the induction system of a piston engine.
B    indicate differential pressure between the intake      Answer (A) is incorrect because a manifold pressure
      manifold and atmospheric pressure.                    gauge has no role in maintaining pressure in the intake
C    indicate absolute pressure in the intake manifold.     manifold, and answer (B) is wrong because a manifold
                                                            pressure gauge indicates absolute pressure, not differ-
                                                            ential pressure.
2-74. H02
The purpose of an exhaust gas analyzer is to indicate the   2-74. Answer B. JSPT 2-10
                                                            The purpose of an exhaust gas analyzer is to indicate
A' brake specific fuel consumption.                         the fuel/air ratio being burned in the cylinders. It identi-
B fuel/air ratio being burned in the cylinders.             fies cylinders that are running too rich or too lean, and
C temperature of the exhaust gases in the exhaust           can be used to fine tune the fuel metering system.
manifold.                                                   Answer (A) is incorrect because brake specific fuel
                                                            consumption is a calculated measure of the amount of
                                                            fuel burned for each horsepower produced and cannot
                                                            be determined by an exhaust gas analyzer. Answer (C)
                                                            is incorrect because an EGT gauge measures the tem-
                                                            perature of the exhaust gases in the exhaust manifold.
2-75.           H02
Which of the following types of electric motors are         2-75. Answer B. JSPT 2-11 (AC 65-15A)
commonly used in electric tachometers?                      A typical electric tachometer consists of a three-phase
                                                            generator mounted to the engine that is connected
A    Direct current, series-wound motors.                   electrically to a three-phase synchronous motor in the
B    Synchronous motors.                                    tachometer instrument. The engine-mounted generator
C    Direct current, shunt-wound motors.                    produces a three-phase current that is sent to the syn-
                                                            chronous motor where a rotating field is produced in the
                                                            stator. The rotating field causes the rotor to turn which,
                                                            in turn, moves the tachometer's indicating needle.
                                                            Answers (A) and (C) are incorrect because these types
                                                            of electric motors are not capable of sending position
                                                            information since they are primarily used for power.
2-18                                                                                                Turbine Engines
2-76.            H02                                        2-76. Answer C. JSPT 2-9 (AC 65-15A) A
Where are the hot and cold junctions located in an          thermocouple is a circuit of two dissimilar metals
engine cylinder temperature indicating system?              connected together at two junctions to form a loop.
                                                            When there is a temperature difference between the
A Both junctions are located at the instrument.             two junctions an electromotive force is produced which
B Both junctions are located at the cylinder.               can be measured with a galvanometer. Therefore,
C The hot junction is located at the cylinder and the       when a thermocouple temperature indicating system is
cold junction is located at the instrument.                 used on a reciprocating engine, the thermocouple's hot
                                                            junction is placed at the cylinder whereas the system's
                                                            cold junction is at the instrument. Answers (A) and (B)
                                                            are incorrect because a thermocouple must have a
                                                            temperature differential in order to produce current flow,
                                                            and there is no temperature difference if both junctions
                                                            are located in the same place.
2-77.            H02                                        2-77. Answer B. JSPT 2-11 (AC 65-15A)
Basically, the indicator of a tachometer system is          The typical electric tachometer system utilizes a
responsive to change in                                     three-phase AC generator coupled to the aircraft engine
                                                            and connected electrically to a synchronous motor
A      current flow.                                        indicator mounted to the instrument panel. The
B      frequency.                                           generator transmits three-phase power to the
C      voltage.                                             synchronous motor at a frequency that is proportional to
                                                            the engine speed. The exact frequency determines the
                                                            motor speed which, in turn, produces the instrument
                                                            indication. Answers (A) and (C) are incorrect because
                                                            changing the current or voltage to the tachometer
                                                            indicator, as long as the values remain within the
                                                            instrument's operating range, has no effect on the
                                                            indicator.
2-78.          H02
Which statement is correct concerning a                     2-78. Answer B. JSPT 2-9 (AC 65-15A)
thermocouple-type temperature indicating instrument         Because a thermocouple produces its own milliamp
system?                                                     current flow, a temperature indicating system using
                                                            thermocouples does not require any external power.
A    It is a balanced-type, variable resistor circuit.      However, most systems do use external power and an
B    It requires no external power source.                  amplifier to improve the response. Answer (A) is incor-
C    It usually contains a balancing circuit in the         rect because a thermocouple produces a temperature
      instrument case to prevent fluctuations of the sys-   indication by generating current, not by changing resis-
      tem voltage from affecting the temperature read-      tance, and answer (C) is wrong because the amplifier
                                                            circuit in a typical indicator removes any transient volt-
                                                            age signals.
2-79.           H02                                         2-79. Answer B. JSPT 2-9 (AC 65-15A)
Which statement is true regarding a thermocouple-type       A thermocouple is a circuit of two dissimilar metals
cylinder head temperature measuring system?                 connected together at two junctions to form a loop.
                                                            When there is a temperature difference between the
A   The resistance required for cylinder head temper-       two junctions an electromotive force is produced which
    ature indicators is measured in farads.                 can be measured with a galvanometer. The greater the
B   The voltage output of a thermocouple system is          temperature difference the greater the voltage pro-
    determined by the temperature difference between        duced. Answer (A) is wrong because resistance is
    the two ends of the thermocouple.                       measured in ohms, not farads, and answer (C) is incor-
C   When the master switch is turned on, a thermocou-       rect because turning on the master switch will have no
    ple indicator will move off-scale to the low side.      effect on a thermocouple indication system.
Turbine Engines                                                                                             2-19
2-80.           H02                                       2-80. Answer B. JSPT 2-9 (AC 65-15A)
What basic meter is used to indicate cylinder head        A thermocouple is a circuit of two dissimilar metals
temperature in most aircraft?                             connected together at two junctions to form a loop.
                                                          When there is a temperature difference between the
A lectrodynamometer.                                      two junctions an electromotive force is produced which
B Galvanometer.                                           can be measured with a galvanometer.
C Thermocouple-type meter.
2-81.           H02                                       2-81. Answer A, JSPT 2-9 (FAR 91.205)
Which of the following is a primary engine instru-        On both reciprocating and turbine powered aircraft, the
ment?                                                     tachometer is the primary engine instrument. On turbo-
                                                          prop and turboshaft powered aircraft that utilize a
A    Tachometer.                                          torque meter gauge, it also becomes a primary engine
B   fuel flowmeter.                                       instrument. Answer (B) is wrong because the only fuel
C   Airspeed indicator.                                   instrument considered a primary instrument is a fuel
                                                          quantity indicator, and answer (C) is incorrect because
                                                          an airspeed indicator is not an engine instrument.
2-82.           H02                                       2-82. Answer A. JSPT 2-7 (AC 65-12A)
A complete break in the line between the manifold         A manifold pressure gauge measures the absolute
pressure gauge and the induction system will be indi-     pressure within an engine's intake manifold. However, if
cated by the gauge registering                            a break exists in the line between the manifold pressure
                                                          gauge and the manifold, the gauge will only be able to
A    prevailing atmospheric pressure.                     display the prevailing atmospheric pressure. Answers
B     zero.                                               (B) and (C) are incorrect because the gauge will still
C    lower than normal for conditions prevailing.         display atmospheric pressure.
2-83.          H02                                        2-83. Answer B. JSPT 2-7 (AC 65-12A)
Engine oil temperature gauges indicate the temperature    In both wet and dry-sump lubricating systems, the oil
of the oil                                                temperature bulb is located somewhere in the oil inlet
                                                          line between the supply tank and the engine. This
A    entering the oil cooler.                             means the oil temperature gauge in the cockpit indi-
B    entering the engine.                                 cates the temperature of the oil entering the engine.
C    in the oil storage tank.                             Neither answer (A) nor (C) is correct because the tem-
                                                          perature of the oil entering the oil cooler and in the
                                                          storage tank are not measured on aircraft.
2-84.           H02                                       2-84. Answer C. JSAT 11-48 (AC 65-15A)
Why do helicopters require a minimum of two               Helicopters require a minimum of two tachometers to
syn- chronous tachometer systems?                         monitor both the engine rpm and the rotor rpm.
                                                          Answers (A) and (B) are wrong because the tail rotor is
A    One indicates engine RPM and the other tail rotor    interconnected with the main rotor and, therefore, its
     RPM.                                                 speed is directly related to the main rotor rpm.
B    One indicates main rotor RPM and the other tail
     rotor RPM.
C    One indicates engine RPM and the other main
     rotor RPM.
2-85.            H02                                      2-85. Answer B. JSPT 2-10 (AC 65-15A)
If the thermocouple leads were inadvertently crossed at   A thermocouple type of temperature indicating system
installation, what would the cylinder temperature         produces a current flow in one direction when there is a
gauge pointer indicate?                                   difference in temperature between the hot junction and
                                                          the cold junction. Therefore, if the leads to the
A   Normal temperature for prevailing condition.          temperature gauge are reversed, the temperature
B   Moves off-scale on the zero side of the meter.        gauge pointer movement will reverse and the needle
C   Moves off-scale on the high side of the meter.        will peg out at the meter's zero side.
2-20                                                                                               Turbine Engines
2-86.          H02                                         2-86. Answer A. JSPT 2-7 (AC 65-15A)
A common type of electrically operated oil temperature     The two types of circuits typically used in electrically
gauge utilizes                                             operated oil temperature gauges are the wheatstone
                                                           bridge circuit and the ratiometer circuit. Answer (B) is
A    either a wheatstone bridge or ratiometer circuit.     incorrect because thermocouple indicators are typically
B    a thermocouple type circuit.                          used to measure cylinder head temperatures and
C    vapor pressure and pressure switches.                 answer (C) is wrong because vapor pressure oil tem-
                                                           perature gauges are not electrically operated.
2-87.           H02                                        2-87. Answer C. JSPT 2-9 (AC 65-12A)
The indication on a thermocouple-type cylinder head        A thermocouple is a circuit of two dissimilar metals
temperature indicator is produced by                       connected together at two junctions to form a loop.
                                                           When there is a temperature difference between the
A    resistance changes in two dissimilar metals.          two junctions an electromotive force is produced which
B    a difference in the voltage between two dissimilar    can be measured with a galvanometer. The greater the
     metals.                                               temperature difference the greater the voltage pro-
C    a current generated by the temperature difference     duced. Answer (A) is wrong because the resistance of
      between dissimilar metal hot and cold junctions.     the dissimilar metals does not change, and answer (B)
                                                           is wrong because there is no voltage "difference"
                                                           between the two dissimilar metals.
2-88.             H02                                      2-88. Answer A. JSGT 14-8 (AC 65-12A)
(1) Powerplant instrument range markings show              Both statements (1) and (2) are correct. Powerplant
    whether the current state of powerplant operation is   range markings show minimum, continuous, limited,
    normal, acceptable for a limited time,                 and maximum ranges. These ranges may not exceed
    or unauthorized.                                       the engine's Type Certificate Data Sheet specifications
(2) Powerplant instrument range markings are based         and, in some cases, may be less than those specified.
    on installed engine operating limits which may not
    exceed (but are not necessarily equal to) those
    limits shown on the engine Type Certificate Data
    Sheet.
Regarding the above statements,
A    both No. 1 and No.2 are true.
B    neither No.l nor No.2 is true.
C    only No.l is true.
2-89.         H02                                          2-89. Answer B. JSPT 2-10 (AC 65-15A)
Thermocouple leads                                         Thermocouple leads are designed for a specific instal-
                                                           lation. For example, in order to function properly there
A   may be installed with either lead to either post of    must be a specific amount of resistance in the thermo-
    the indicator.                                         couple circuit. Thus, their length or cross sectional size
B   are designed for a specific installation and may       cannot be altered unless some compensation is made
    not be altered.                                        for the change in total resistance. A thermocouple sys-
C   may be repaired using solderless connectors.           tem for a typical turbine engine, for example, has eight
                                                           ohms of resistance. Answer (A) is incorrect because
                                                           thermocouple leads are polarized and can only be
                                                           installed one way. Answer (C) is wrong because ther-
                                                           mocouple leads have precise resistances that are
                                                           matched to each installation and cannot be repaired.
Turbine Engines                                                                                              2-21
2-90.            H02                                    2-90. Answer A. JSPT 2-11 (AC 65-15A)
What unit in a tachometer system sends information to   The typical electric tachometer system uses a
the indicator?                                          three-phase AC generator coupled to the engine to
                                                        send information to an AC synchronous motor that is
A    The three-phase ac generator.                      attached to the indicator. Answer (B) is wrong because
B    The two-phase ac generator.                        two-phase AC is not used in aircraft tachometers.
C    The synchronous motor.                             Answer (C) is incorrect because a synchronous motor
                                                        is located in the tachometer indicator and receives the
                                                        signal sent by a three-phase AC generator.
2-91.           H02                                     2-91. Answer B. JSPT 2-12 (AC 65-15A)
Which of the following instrument discrepancies         Items 2, 4, 5 and 8 cannot be repaired by a mainte-
require replacement of the instrument?                  nance technician and, therefore, the instrument must
                                                        be removed from the aircraft and sent to a certified
1. Red line missing from glass.                         instrument repair station. The remaining items (items 1,
2. Glass cracked.                                       3, 6, and 7) can be repaired by an aviation technician
3. Case paint chipped.                                  and do not require the removal and replacement of the
4. Will not zero out.                                   instrument. Therefore, answer (B) is correct.
5. Pointer loose on shaft.
6. Mounting screw loose.
7. Leaking at line B nut.
8. Fogged.
A 2, 3,7, 8.
B 2,4, 5, 8.
C 1,2,4, 7.
2-92.         H02                                       2-92. Answer A. JSPT 2-8 (AC 65-15A)
A Bourdon-tube instrument may be used to indicate       A Bourdon tube is a metal tube that is formed in a circular
                                                        shape with an oval or flattened cross section and is used
1. pressure.                                            to measure both pressure and temperature (answer A).
2. temperature.                                         When air or liquid pressure enters the open end of a
3. position.                                            Bourdon tube, the tube has a tendency to straighten out.
4. quantity.                                            By the same token, if a Bourdon tube is filled with a gas
                                                        and sealed at both ends, changes in temperature will
A 1 and 2..                                             cause the sealed gas to expand and contract thereby
B 1and 3.                                               causing the tube to move. Through a series of gears, this
C 2 and 4.                                              movement is then used to move an indicating needle.
2-93.           H02                                     2-93. Answer C. JSPT 2-13 (AC 65-15A)
Which of the following instrument conditions is         Chipped paint on an instrument case has no effect on
acceptable and does NOT require immediate               an instrument's operational condition and, therefore, no
correction?                                             immediate corrective action is required. Answer (A) is
                                                        incorrect because a missing red line (item 1) could
1. Red line missing.                                    threaten operating safety and answer (B) is wrong
2. Pointer loose on shaft.                              because loose mounting screws (item 4) could subject
3. Glass cracked.                                       the instrument to vibration damage. Both items require
4. Mounting screws loose.                               immediate attention.
5. Case paint chipped.
6. Leaking at line B nut.
7. Will not zero out.
8. Fogged.
A 1.
B 4.
C 5.
2-22                                                                                              Turbine Engines
2-94.            H02                                       2-94. Answer C. JSPT 2-7 (AC 65-15A)
Instruments that provide readings of low or negative       Typically, instruments that provide low or negative
pressure, such as manifold pressure gauges, are usually    pressure readings utilize a sensitive diaphragm or bel-
what type?                                                 lows that expands and contracts to drive an indicator
                                                           needle. Answer (A) is incorrect because vanes are typ-
A    Vane with calibrated spring.                          ically used to measure volume flow or high pressure air
B    Bourdon tube.                                         flow. Answer (B) is incorrect because the shape of a
C    Diaphragm or bellows.                                 Bourdon tube does not allow it to accurately indicate
                                                           low or negative pressures.
2-95.             H02                                      2-95. Answer B. JSPT 2-8 (AC 65-15A)
Instruments that measure relatively high fluid pres-       Oil pressure gauges typically utilize a Bourdon tube
sures, such as oil pressure gauges, are usually what       type indicator. A Bourdon tube is a metal tube that is
type?                                                      formed in a circular shape with an oval or flattened
                                                           cross section. When air or liquid pressure enters the
A    Vane with calibrated spring.                          open end of a Bourdon tube, the tube has a tendency to
B    Bourdon tube.                                         straighten out. Through a series of gears, this move-
C    Diaphragm or bellows.                                 ment is used to move an indicating needle. Answer (A)
                                                           is incorrect because vane type transmitters are used in
                                                           fluid flow systems and measure volume of flow. Answer
                                                           (C) is wrong because diaphragm or bellows type indi-
                                                           cators are generally used to measure relatively low air
                                                           pressures.
2-96.         H02                                          2-96. Answer C. JSPT 2-11 (AC 65-15A)
The RPM indication of a synchronous ac                     Modern AC tachometers utilize a three-phase AC gen-
motor-tachometer is governed by the generator              erator coupled to an engine and connected electrically
                                                           to an indicator mounted in the instrument panel. As an
A    voltage.                                              engine runs, the three-phase generator transmits a fre-
B    current.                                              quency that is proportional to the engine speed of a
C    frequency.                                            synchronous motor mounted to the indicator. The
                                                           transmitted frequency causes the synchronous motor to
                                                           turn at a specific rpm and provide a specific indication.
                                                           Answers (A) and (B) are incorrect because changing
                                                           voltage or current will have no effect on a synchronous
                                                           motor tachometer, providing the values of current or
                                                           voltage do not exceed the operating limits of the
                                                           instrument.
2-97.           H02                                        2-97. Answer A. JSPT 2-10 (AC 65-15A)
The EGT gauge used with reciprocating engines is           The exhaust gas temperature (EGT) gauge provides a
primarily used to furnish temperature readings in order    pilot with a means of properly adjusting the fuel/air mix-
to                                                         ture for efficient operation. Answers (B) and (C) are
                                                           incorrect because the oil temperature and cylinder head
A   obtain the best mixture setting for fuel efficiency.   temperature gauges are typically used to monitor engine
B   obtain the best mixture setting for engine cooling.    cooling and prevent an overtemperature condition.
C   prevent engine overtemperature.
                                                           2-98. Answer B. JSPT 2-10 (AC 20-88A)
                                                           A red mark on an instrument face designates the maxi-
2-98.            H02                                       mum limit for a high transient condition. Answer (A) is
A red triangle, dot, or diamond mark on an engine          incorrect because a red radial line is used to indicate a
instrument face or glass indicates                         maximum operating limit, and answer (C) is wrong
                                                           because a restricted operating range is indicated by a
A   the maximum operating limit for all normal oper-       yellow arc.
     ations.
B   the maximum limit for high transients such as
     starting.
C   a restricted operating range.
Turbine Engines                                                                                                     2-23
2-99.         L02                                            2-99. Answer C. JSPT 2-9 (AC 65-12A)
Thermocouples are usually inserted or installed on the       A thermocouple-type indicating device is typically used to
                                                             indicate the cylinder head temperature on a reciprocating
A    front cylinder of the engine.                           engine. To help ensure that none of the cylinders are hot-
B    rear cylinder of the engine.                            ter than the temperature indicated in the cockpit, cylinder
C    hottest cylinder of the engine.                         head temperature readings are usually taken from the
                                                             hottest cylinder. Temperature readings are not taken at the
                                                             front cylinder of an engine (answer A) because these
                                                             cylinders receive a direct blast of cooling air and, therefore,
                                                             are not representative of actual operating conditions.
                                                             Although a rear cylinder (answer B) is typically the hottest
                                                             cylinder on most four cylinder engines, this may not be the
                                                             case with a six or eight cylinder engine.
2-100.           L02                                         2-100. Answer C. JSPT 2-28 (AC 65-12A)
A spark plug is fouled when                                  A spark plug is fouled when it becomes contaminated
                                                             with foreign matter to the point that the spark flows
A    its spark grounds by jumping electrodes.                through the foreign matter to ground rather than jumping
B    it causes preignition.                                  the air gap at the electrode. Answer (A) is incorrect
C    its spark grounds without jumping electrodes.           because a normally operating spark plug grounds by
                                                             jumping electrodes. Answer (B) is wrong because
                                                             preignition occurs when an isolated hot spot in the
                                                             cylinder ignites the fuel/air mixture before the time of
                                                             normal ignition.
2-101.           M04                                         2-101. Answer B. JSPT 2-18 (AC 65-12A)                 •
Under which of the following conditions would an             To ensure efficient combustion, fuel must be properly
engine run lean even though there is a normal amount         mixed with air, or atomized, before it enters the cylin-
of fuel present?                                             ders. The more fully a mixture is vaporized, the greater
                                                             the efficiency of the combustion process. On the other
A The use of too high an octane rating fuel.                 hand, if the fuel is not fully vaporized, less fuel mixes
B Incomplete fuel vaporization.                              with the intake air and the mixture becomes lean even
C The carburetor air heater valve in the HOT posi-           though there is an abundance of fuel present. Answer
tion.                                                        (A) is incorrect because the use of a fuel with an
                                                             excessively high octane rating will have no effect on the
                                                             fuel/air mixture. Answer (C) is wrong because the
                                                             application of carburetor heat introduces less dense air
                                                             into the engine which essentially enriches the mixture.
2-102.          N01                                          2-102. Answer C. JSPT 2-14 (AC 65-9A)
When an electric primer is used, fuel pressure is built      One of the many purposes of a boost pump is to supply
up by the                                                    fuel under pressure for priming prior to starting an
                                                             engine. Answer (A) is wrong because the primer sole-
A.   internal pump in the primer solenoid.                   noid does not contain its own pump, and answer (B) is
B.   suction at the main discharge nozzle.                   incorrect because, when the engine is not running,
C.   booster pump.                                           there is no suction at the main discharge nozzle.
2-103.              O03                                      2-103. Answer B. JSPT 2-14 (AC 65-9A)
If a fire starts in the induction system during the engine   If a fire breaks out in a reciprocating engine's induction
starting procedure, what should the operator do?             system during a start attempt, you should continue to crank
                                                             the engine to try and draw the fire into the engine (answer
A    Turn off the fuel switches to stop the fuel.            B). If the fire does not go out, the fuel selector valve and
B    Continue cranking the engine.                           ignition should be shut off, the mixture placed in the idle
C    Turn off all switches.                                  cutoff position, and a fire extinguisher used to put out the
                                                             fire. Answers (A) and (C) are wrong because the first thing
                                                             you should do is continue cranking the engine.
2-104.         P02
An engine becomes overheated due to excessive taxiing or improper ground runup. Prior to
shutdown, operation must continue until cylinders have cooled, by running engine at
A    low RPM with oil dilution system activated.
B    idle RPM.
C    high RPM with mixture control in rich position.
2-105.          P02
High cylinder head temperatures are likely to result from
A    a very lean mixture at high power settings.
B    fouled spark plugs.
C    a very rich mixture at high power settings.
2-106.          A02
Grinding the valves of a reciprocating engine to a feather edge is likely to result in
A    normal operation and long life.
B    excessive valve clearance.
C    preignition and burned valves.
2-104. Answer B. JSPT 2-17 (AC 65-12A)
After a flight and a few minutes of taxiing, an engine typically will not become excessively warm and,
therefore, can be shut down almost immediately. However, if an engine becomes excessively hot
as indicated by the cylinder head temperature gauge and the oil temperature gauge, you should
allow the engine to cool at idle speed for a short time before shutdown. Answer (A) is wrong
because an engine's oil dilution system reduces oil viscosity to facilitate starting in cold weather and
should never be used when the engine is overheated. Answer (C) is incorrect because operating an
overheated engine at high rpm will only make the problem worse.
2-105. Answer A. JSPT 2-17 (AC 65-12A)
At higher power settings, a very lean mixture does not allow any excess fuel into the engine to
aid in cooling and, therefore, high cylinder temperatures typically result. Answer (B) is wrong
because fouled spark plugs typically fire intermittently and result in lower cylinder temperatures.
Answer (C) is incorrect because the excess fuel in a very rich mixture helps cool engine
cylinders and maintain low cylinder head temperatures.
2-106. Answer C. JSPT 2-58 (AC 65-12A)
A thin edge on a poppet valve is called a feather edge. Valves with a feather edge are likely to overheat
and burn away in a short period of time. Both of these conditions can lead to preignition. Answer (A) is
incorrect because normal operation and long life are not characteristics of valves with a feather edge.
Answer (B) is wrong because the type of edge on a valve does not affect valve clearance.